Produit, logarithme et intégrales et équivalent en l'infini


Colle de mathématiques

Sujet de colle de maths:

Énoncé du sujet

  1. Calculer l'intégrale $\dsp\int_1^e \ln t\,dt$
  2. Montrer que, pour tout $i\geq 2$,    $\dsp\int_{i-1}^i\ln t\,dt\leq\ln i\leq\int_i^{i+1}\ln t \,dt$.
  3. Montrer que, pour tout entier $n\geq 1$,    $\dsp\int_1^n \ln t\,dt\leq \ln(n!)\leq \int_1^n\ln t \,dt+\ln n$.
  4. En déduire que $\ln(n!)$ est équivalent à $n\ln(n)$ lorsque $n$ tend vers $+\infty$.



Correction

Correction

  1. On réalise une intégration par parties, en écrivant $\ln(t)=1\times \ln(t)$, d'où

    \[\begin{array}{ll}\dsp\int_1^e\ln(t)dt
  &=\Bigl[ t\ln(t)\Bigr]_1^e-\dsp\int_1^e t\tm\dfrac1tdt\\[1em]
  &=e\ln e-e+1
  =1\enar\]

  2. Soit $t\in [i-1,i]$. Alors, puisque la fonction logarithme est croissante, on a

    \[\ln(t)\leq \ln(i)\]

    et donc, en intègrant sur $[i-1,i]$,

    \[\int_{i-1}^i \ln(t)dt\leq\int_{i-1}^i\ln(i)dt=(i-(i-1))\ln(i)=\ln(i)\]

    La deuxième partie de l'inégalité se prouve exactement de la même façon, en remarquant que pour tout $t$ dans $[i,i+1]$, on a
    \[\ln(i)\leq\ln(t)\]

  3. On commence par l'inégalité de gauche, en sommant membre à membre l'inégalité de gauche précédente pour $i$ allant de $2$ jusqu'à $n$.
    Par la formule de Chasles, le membre de gauche est alors

    \[\int_1^2\ln(t)dt+\int_2^3\ln(t)dt+\dots+\int_{n-1}^n\ln(t)dt=\int_1^n \ln (t)dt\]

    Le membre du milieu vaut lui
    \[\sum_{i=2}^n \ln(i)=\ln\left(\prod_{i=2}^n i\right)=\ln(n!)\]



    On s'occupe ensuite de l'inégalité de droite. On somme membre à membre l'inégalité de droite de la question précédente, mais cette fois pour $i$ de 2 à $n$.
    On obtient
    \[\ln\big((n-1)!\big)\leq \int_1^n\ln(t)dt\]

    Il suffit ensuite d'ajouter $\ln(n)$ de chaque côté de l'inégalité pour obtenir le résultat demandé.
  4. Des deux questions précédentes, on tire

    \[n\ln n-n+1\leq\ln(n!)\leq n\ln n+\ln n-n+1\]

    soit encore
    \[1-\dfrac1{\ln n}+\dfrac1{n\ln n}
  \leq\frac{\ln(n!)}{n\ln n}
  \leq 1+\dfrac1n-\dfrac1{\ln n}+\dfrac1{n\ln n}\]

    Par le théorème des gendarmes, on trouve alors que
    \[\lim_{n\to+\infty}\dfrac{\ln(n!)}{n\ln (n)}=1\]

    Ceci signifie exactement que $\ln(n!)$ est équivalent à $n\ln(n)$ lorsque $n$ tend vers $+\infty$.


Tag:Intégrale

Autres sujets au hasard: Lancer de dés
LongPage: h2: 3 - h3: 0